Đến nội dung

Hình ảnh

Tổng hợp các bài BĐT trong các đề thi thử THPT Quốc Gia môn Toán năm 2017


  • Please log in to reply
Chủ đề này có 383 trả lời

#41
tungteng532000

tungteng532000

    Trung sĩ

  • Thành viên
  • 174 Bài viết

Bài 24:Cho $x,y,z$ là các số thực không âm thỏa mãn $x^{2}+y^{2}+z^{2}=3$. Tìm GTNN của 

   $P=\frac{16}{\sqrt{x^{2}y^{2}+y^{2}z^{2}+z^{2}x^{2}+1}}+\frac{xy+yz+zx+1}{x+y+z}$

 

_Đề thi thử Chu Văn An Sơn La_

Bạn dùng bđt này là làm đc nhé: $\sum x\geq \sum x^2y^2$ với $\sum x^2=3$


                                              Lời giải hay thì like nhé :))
FB: 
https://www.facebook...oylanh.lung.564


#42
tungteng532000

tungteng532000

    Trung sĩ

  • Thành viên
  • 174 Bài viết

Sory mọi người mấy hôm nay đang ôn thi nên bận quá

 

Bài 25 (Đề THPT Lý Thái Tổ - Bắc Ninh - Lần 3 -2016)

 

Cho $a,b,c$ là ba số thực dương thỏa mãn $a+b+c=1$.Tìm Min

 

$P=\frac{ac}{b+ac}+\frac{b}{a+bc}+\frac{9(a^3+b^3+c^3+12c^2+12(ab+bc+ac)+a+b+c)}{6(c+ab)}$

bạn viết đề sai thì phải :)


                                              Lời giải hay thì like nhé :))
FB: 
https://www.facebook...oylanh.lung.564


#43
niasco

niasco

    Lính mới

  • Thành viên mới
  • 7 Bài viết

$\boxed{3}$ ( Đề lần 2-2016- Chuyên Vĩnh phúc)

 

Cho $a,b,c$ là độ dài 3 cạnh tam giác có chu vi là 1 . Tìm giá trị nhỏ nhất của :

 

$$T=\frac{4}{a+b}+\frac{4}{b+c}+\frac{4}{a+c}-\frac{1}{a}-\frac{1}{b}-\frac{1}{c}$$

 

 

 

 

Bạn xem lại giúp mình, theo mình là Giá trị lớn nhất của T thì đúng hơn

Giải.

Vì a,b,c là độ dài 3 cạnh tam giác nên 2a,2b,2c < $a+b+c=1$ hay 0 < a,b,c <$\frac{1}{2}$

Ta có :

$\frac{4}{1-x} -\frac{1}{x} \leq $18x-3$\Leftrightarrow (2x-1)(3x-1)^{2}\leq 0$ (luôn đúng)

$\Rightarrow T\leq 18(a+b+c) -3.3= 9$
Vậy max T là 9 tại $a=b=c=\frac{1}{3}$


Bài viết đã được chỉnh sửa nội dung bởi niasco: 11-04-2016 - 10:54


#44
Dinh Xuan Hung

Dinh Xuan Hung

    Thành viên nổi bật 2015

  • Thành viên nổi bật 2016
  • 1396 Bài viết

Sory mọi người mấy hôm nay đang ôn thi nên bận quá

 

Bài 25 (Đề THPT Lý Thái Tổ - Bắc Ninh - Lần 3 -2016)

 

Cho $a,b,c$ là ba số thực dương thỏa mãn $a+b+c=1$.Tìm Min

 

$P=\frac{ac}{b+ac}+\frac{b}{a+bc}+\frac{9(a^3+b^3+c^3)+12c^2+12(ab+bc+ac)+a+b+c}{6(c+ab)}$

 

Ta có:$P=\frac{ac}{ac+b}+\frac{bc}{bc+a}+\frac{9(a^3+b^3+c^3)+1}{6(c+ab)}+2$

 

Áp dụng bổ đề: $\frac{1}{x+1}+\frac{1}{y+1}\geq \frac{2}{\sqrt{xy}+1} (xy\geq 1)$

 

Khi đó ta có:$\frac{ac}{ac+b}+\frac{bc}{bc+a}\geq \frac{2c}{1+c}$

 

Đồng thời thì $9(a^3+b^3+c^3)+1\geq 6\sqrt{a^3+b^3+c^3}=6\sqrt{(a+b+c)(a^3+b^3+c^3)}\geq 6(a^2+b^2+c^2)$

 

Lại có:$(a^2+b^2)\geq \frac{(a+b)^2}{2};ab\leq \frac{(a+b)^2}{4}$

 

Thay a+b=1-c vào thì được :$P\geq f(c)=\frac{2c}{c+1}+\frac{2(1-c)^2+4c^2}{4c+(1-c)^2}+2\geq \frac{5}{4}+2=\frac{13}{4}$



#45
Dinh Xuan Hung

Dinh Xuan Hung

    Thành viên nổi bật 2015

  • Thành viên nổi bật 2016
  • 1396 Bài viết

Bài 26 : (THPT Quốc Học Huế-Thi thử lần 2)

 

Cho $x,y,z \in [1;3]$ thỏa mãn $y+z-4x=0$.Tìm giá trị lớn nhất và nhỏ nhất của biểu thức:

 

$$P=\frac{y^2+z^2-4x^2}{\sqrt{x^2y^2+y^2z^2+z^2x^2-7x^4}}$$ 

 

Bài 27 (Đề thi thử môn Toán năm 2016 trường THPT ĐăkMil, ĐăkNông)

 

Cho $a,b,c$ là các số thực không âm thỏa mãn:$ab+bc+ac=1$.Tìm Min biểu thức:

 

$$P=\sqrt{\frac{a}{16(b+c)(a^2+bc)}}+\sqrt{\frac{b}{16(a+c)(b^2+ac)}}+\frac{a^2+1}{4}\left ( \frac{1}{a}+\frac{c}{ab} \right )$$


Bài viết đã được chỉnh sửa nội dung bởi Dinh Xuan Hung: 16-04-2016 - 20:25


#46
phamngochung9a

phamngochung9a

    Sĩ quan

  • Điều hành viên THPT
  • 480 Bài viết

Bài 26 : (THPT Quốc Học Huế-Thi thử lần 2)

 

Cho $x,y,z \in [1;3]$ thỏa mãn $y+z-4x=0$.Tìm giá trị lớn nhất và nhỏ nhất của biểu thức:

 

$$P=\frac{y^2+z^2-4x^2}{\sqrt{x^2y^2+y^2z^2+z^2x^2-7x^4}}$$ 

Từ giả thiết, ta có: $\frac{y}{x}+\frac{z}{x}=4$

$P=\frac{\left ( \frac{y}{x} \right )^{2}+\left ( \frac{z}{x} \right )^{2}-4}{\sqrt{\left ( \frac{y}{x} \right )^{2}+2.\left ( \frac{y}{x} \right )^{2}.\left ( \frac{z}{x} \right )^{2}+\left ( \frac{z}{x} \right )^{2}-7}}$

Đặt $\left\{\begin{matrix} \frac{y}{x}=a & \\ \frac{z}{x}=b & \end{matrix}\right.\Rightarrow a+b=4$

$P=\frac{a^{2}+b^{2}-4}{\sqrt{a^{2}+a^{2}b^{2}+b^{2}-7}}=\frac{12-2t}{\sqrt{9-2t+t^{2}}}$ (với $t=ab$)

Đặt $f\left ( t \right )=\frac{12-2t}{\sqrt{t^{2}-2t+9}}$, ta có: $f'\left ( t \right )< 0$

$\Rightarrow f\left ( t \right )$ nghịch biến.

Ta sẽ giới hạn giá trị của $t$. Ta có:

*) $t=\frac{y}{x}.\frac{z}{x}\leq \frac{1}{4}\left ( \frac{y}{x}+\frac{z}{x} \right )^{2}=4$

Vậy $f\left ( t \right )\geq f\left ( 4 \right )=\frac{4}{\sqrt{17}}$

*) $a\in \left [ \frac{1}{3};3 \right ]$, $b\in \left [ \frac{1}{3};3 \right ]$

$\Rightarrow \left ( a-3 \right )\left ( b-3 \right )\geq 0\Rightarrow ab\geq 3a+3b-9=3\Rightarrow t\geq 3$

Vậy $f\left ( t \right )\leq f\left ( 3 \right )=\sqrt{3}$

 

 

        Answer: $\min P=\frac{4}{\sqrt{17}}\Leftrightarrow y=z=2x$, chẳng hạn: $y=z=2x=2$

                      $\max P=\sqrt{3}$, khi chẳng hạn: $\left\{\begin{matrix} x=1 & & \\ y=3 & & \\ z=1 & & \end{matrix}\right.$



#47
phamngochung9a

phamngochung9a

    Sĩ quan

  • Điều hành viên THPT
  • 480 Bài viết

Bài 28: (Đề thi thử của sở GD- ĐT Thanh Hóa năm 2016)

Cho $a,b,c$ là độ dài ba cạnh của một tam giác. Tìm giá trị lớn nhất của biểu thức:

$P=\left ( a+b+c \right )\left ( \frac{3a-b}{a^{2}+ab}+\frac{3b-c}{b^{2}+bc}+\frac{3c-a}{c^{2}+ca} \right )$

 

Bài 29: (Đề thi thử của sở GD- ĐT Bắc Giang năm 2016)

Cho ba số thực dương $x,y,z$ thỏa mãn $xy+yz+zx+xyz=4$. Chứng minh rằng:

$3\left ( \frac{1}{\sqrt{x}}+\frac{1}{\sqrt{y}}+\frac{1}{\sqrt{z}} \right )^{2}\geq \left ( x+2 \right )\left ( y+2 \right )\left ( z+2 \right )$

 

Bài 30: (THPT chuyên Thái Bình- lần 3)

Cho ba số thực dương $x,y,z$ thỏa mãn $x+2y+3z=1$. Tìm giá trị lớn nhất của biểu thức:

$P=x^{2}\left ( 5-6x \right )+4y^{2}\left ( 5-12y \right )+z^{2}\left ( 45-162z \right )$


Bài viết đã được chỉnh sửa nội dung bởi Dinh Xuan Hung: 18-04-2016 - 15:47


#48
Dinh Xuan Hung

Dinh Xuan Hung

    Thành viên nổi bật 2015

  • Thành viên nổi bật 2016
  • 1396 Bài viết

 

Bài 29: (Đề thi thử của sở GD- ĐT Bắc Giang năm 2016)

Cho ba số thực dương $x,y,z$ thỏa mãn $xy+yz+zx+xyz=4$. Chứng minh rằng:

$3\left ( \frac{1}{\sqrt{x}}+\frac{1}{\sqrt{y}}+\frac{1}{\sqrt{z}} \right )^{2}\geq \left ( x+2 \right )\left ( y+2 \right )\left ( z+2 \right )$

 

 

1.PNG


Bài viết đã được chỉnh sửa nội dung bởi Dinh Xuan Hung: 16-04-2016 - 08:34


#49
Juliel

Juliel

    Thượng úy

  • Thành viên
  • 1240 Bài viết

 

Bài 27 (Đề thi thử môn Toán năm 2016 trường THPT ĐăkMil, ĐăkNông)

 

Cho $a,b,c$ là các số thực không âm thỏa mãn:$ab+bc+ac=1$.Tìm Min biểu thức:

 

$$P=\sqrt{\frac{a}{16(b+c)(a^2+bc)}}+\sqrt{\frac{b}{16(a+c)(b^2+ac)}}+\frac{a^2+1}{4}\left ( \frac{1}{a}+\frac{c}{ab} \right )$$

Lời giải :

 

Theo AM-GM :

$$\sqrt{\dfrac{a}{(b+c)(a^2+bc)}}=\dfrac{a}{\sqrt{(ab+ac)(a^2+bc)}}\geq \dfrac{a}{\dfrac{1}{2}(ab+ac+a^2+bc)}=\dfrac{2a}{(a+b)(a+c)}$$

Tương tự :

$$\sqrt{\dfrac{b}{(a+c)(b^2+ac)}}\geq \dfrac{2b}{(b+c)(b+a)}$$

Suy ra :

$$4P\geq \dfrac{2a}{(a+b)(a+c)}+\dfrac{2b}{(b+c)(b+a)}+\dfrac{(a^2+1)(b+c)}{ab}=\dfrac{2a(b+c)+2b(c+a)}{(a+b)(b+c)(c+a)}+\dfrac{(a^2+ab+bc+ca)(b+c)}{ab}$$

$$=\dfrac{2(1+ab)}{(a+b)(b+c)(c+a)}+\dfrac{(a+b)(b+c)(c+a)}{ab}\geq 2\sqrt{\dfrac{2(1+ab)}{ab}}=2\sqrt{\dfrac{4(1+ab)}{2ab}}\geq 2\sqrt{\dfrac{4(ab+1)}{1+ab}}=4$$

(Chú ý rằng $ab\leq ab+bc+ca=1$)

Từ đó ta có $MinP=1\Leftrightarrow a=b=1,c=0$


Bài viết đã được chỉnh sửa nội dung bởi Juliel: 16-04-2016 - 20:28

Đừng rời xa tôi vì tôi lỡ yêu người mất rồi !
 

Welcome to My Facebook !


#50
Juliel

Juliel

    Thượng úy

  • Thành viên
  • 1240 Bài viết

Bài 30: (THPT chuyên Thái Bình- lần 3)

Cho ba số thực dương $x,y,z$ thỏa mãn $x+2y+3z=4$. Tìm giá trị lớn nhất của biểu thức:

$P=x^{2}\left ( 5-6x \right )+4y^{2}\left ( 5-12y \right )+z^{2}\left ( 45-162z \right )$

Lời giải :

 

Đặt $a=x,b=2y,c=3z$ thì $a+b+c=4$. Khi đó :

$$P=a^2(5-6a)+b^2(5-6b)+c^2(5-6c)=5a^2+5b^2+5c^2-6a^3-6b^3-6c^3$$

Bằng phương pháp tiếp tuyến, chỉ ra được :

$$5a^2-6a^3\leq \dfrac{-56}{3}a+\dfrac{176}{9}$$

Suy ra :

$$P\leq \dfrac{-56}{3}(a+b+c)+\dfrac{176}{3}=-16$$

$$MaxP=-16\Leftrightarrow x=\dfrac{4}{3},y=\dfrac{2}{3},z=\dfrac{4}{9}$$


Đừng rời xa tôi vì tôi lỡ yêu người mất rồi !
 

Welcome to My Facebook !


#51
niasco

niasco

    Lính mới

  • Thành viên mới
  • 7 Bài viết

Giả sử $y$ nằm giữa $x$ và $z$, ta có:

$(y-z)(y-x)\leq 0\\\Rightarrow y^{2}+xz\leq xy+zy\\\Rightarrow y^{2}z+z^{2}x\leq xyz+z^{2}y\\\Rightarrow P\leq x^{2}y+xyz+z^{2}y=y(x^{2}+zx+z^{2})\leq y(x+z)^{2}=\frac{1}{2}.2y(x+z).(x+z)\leq \frac{1}{54}(2x+2y+2z)^{3}=4$

Vậy $\max P=4\Leftrightarrow \left\{\begin{matrix} x=2 & & \\ y=1 & & \\ z=0 & & \end{matrix}\right.$

Cho mình hỏi tại sao lại giả sử y nằm giữa x và z vậy ? Vì mình thấy đa số bài toán đều giả sử y= min{x,y,z} hoặc y= max{x,y,z} khi các biến hoán vị  :mellow:



#52
ineX

ineX

    Sĩ quan

  • Thành viên
  • 353 Bài viết

Bài 31: (THPT Hàn Thuyên lần 1)

Cho các số thực $x,y,z$ thỏa $x>2$, $y>1$, $z>0$

Tìm giá trị lớn nhất của:

 

$P= \frac{1}{\sqrt{x^{2}+y^{2}+z^{2}-2(2x+y-3)}}-\frac{1}{y(x-1)(z+1)}$


Bài viết đã được chỉnh sửa nội dung bởi Dinh Xuan Hung: 19-04-2016 - 20:49

"Tôi sinh ra là để thay đổi thế giới chứ không phải để thế giới thay đổi tôi" - Juliel

 

3cf67218ea144a6eb6caf571068071ff.1.gif


#53
phamngochung9a

phamngochung9a

    Sĩ quan

  • Điều hành viên THPT
  • 480 Bài viết

Cho mình hỏi tại sao lại giả sử y nằm giữa x và z vậy ? Vì mình thấy đa số bài toán đều giả sử y= min{x,y,z} hoặc y= max{x,y,z} khi các biến hoán vị  :mellow:

Đối với các bài toán hoán vị, ta được phép giả sử vị trí của một biến bất kì, chứ không bắt buộc phải giả sử là lớn nhất hay nhỏ nhất đâu bạn. Ở đây, gs lớn nhất là vị trí đầu, gs nhỏ nhất là vị trí cuối.

 

Lời giải :

 

Đặt $a=x,b=2y,c=3z$ thì $a+b+c=4$. Khi đó :

$$P=a^2(5-6a)+b^2(5-6b)+c^2(5-6c)=5a^2+5b^2+5c^2-6a^3-6b^3-6c^3$$

Bằng phương pháp tiếp tuyến, chỉ ra được :

$$5a^2-6a^3\leq \dfrac{-56}{3}a+\dfrac{176}{9}$$

Suy ra :

$$P\leq \dfrac{-56}{3}(a+b+c)+\dfrac{176}{3}=-16$$

$$MaxP=-16\Leftrightarrow x=\dfrac{4}{3},y=\dfrac{2}{3},z=\dfrac{4}{9}$$

Xin lỗi anh nha, tại em hấp tấp gõ sai đề. Lúc đầu em cũng nghĩ dùng UTC nhưng sau đó thất bại, thấy anh làm PP này lại được kiểm tra lại đề thì.....Bây giờ thì không dùng UTC được đâu anh  :P (em nghĩ vậy)


Bài viết đã được chỉnh sửa nội dung bởi phamngochung9a: 17-04-2016 - 09:19


#54
phamngochung9a

phamngochung9a

    Sĩ quan

  • Điều hành viên THPT
  • 480 Bài viết

Bài 30: (THPT chuyên Thái Bình- lần 3)

Cho ba số thực dương $x,y,z$ thỏa mãn $x+2y+3z=1$. Tìm giá trị lớn nhất của biểu thức:

$P=x^{2}\left ( 5-6x \right )+4y^{2}\left ( 5-12y \right )+z^{2}\left ( 45-162z \right )$

Đặt $\left\{\begin{matrix} x=a & & \\ y=2b & & \\ z=3c & & \end{matrix}\right.$

Dễ nhận thấy BĐT đạt điểm rơi tại $a=b=c=\frac{1}{3}$ hoặc $a=b=\frac{1}{2};c=0$ điều này khiến ta liên tưởng đến BĐT Schur.

Đồng bậc hai vế, ta cần chứng minh:

$5\left ( a+b+c \right )\left (a^{2}+b^{2}+c^{2} \right )-6\left ( a^{3}+b^{3}+c^{3} \right )\leq \left ( a+b+c \right )^{3}\\\Leftrightarrow a^{3}+b^{3}+c^{3}+3abc\geq ab\left ( a+b \right )+bc\left ( b+c \right )+ca\left ( c+a \right )$

Vì đây là đề thi THPT QG nên sẽ phải cm lại BĐT Schur. Cách chứng minh ngắn nhất là sử dụng tính chất đối xứng.

Gs $a\geq b\geq c\geq 0$, ta có:

$a^{3}+b^{3}+c^{3}+3abc-ab\left ( a+b \right )-bc\left ( b+c \right )-ca\left ( c+a \right )\\=a\left ( a-b \right )\left ( a-c \right )+b\left ( b-a \right )\left ( b-c \right )+c\left ( c-a \right )\left ( c-b \right )\\\geq a\left ( a-b \right )\left ( b-c \right )+b\left ( b-a \right )\left ( b-c \right )+c\left ( a-c \right )\left ( b-c \right )\\=\left ( a-b \right )^{2}\left ( b-c \right )+c\left ( a-c \right )\left ( b-c \right )\geq 0\left ( TRUE \right )$

Vậy $max P= 1$ khi........ copy ở trên


Bài viết đã được chỉnh sửa nội dung bởi phamngochung9a: 17-04-2016 - 09:24


#55
Juliel

Juliel

    Thượng úy

  • Thành viên
  • 1240 Bài viết

Bài 32 (Thi thử THPT Quốc gia 2016 THPT Chuyên Lương Thế Vinh, Đồng Nai)

Cho $a,b,c$ dương thoả $\dfrac{1}{a}+\dfrac{1}{b}+\dfrac{1}{c}=3$. Tìm GTNN :

$$P=(a+1)(b+1)(c+1)+\dfrac{4}{\sqrt{a^2+b^2+c^2+1}}$$


Bài viết đã được chỉnh sửa nội dung bởi Dinh Xuan Hung: 19-04-2016 - 20:49

Đừng rời xa tôi vì tôi lỡ yêu người mất rồi !
 

Welcome to My Facebook !


#56
quanguefa

quanguefa

    Thiếu úy

  • Thành viên
  • 596 Bài viết

Bài 31: (THPT Hàn Thuyên lần 1)

Cho các số thực $x,y,z$ thỏa $x>2$, $y>1$, $z>0$

Tìm giá trị lớn nhất của:

 

$P= \frac{1}{\sqrt{x^{2}+y^{2}+z^{2}-2(2x+y-3)}}-\frac{1}{y(x-1)(z+1)}$

Bài 31: Dạng này xuất hiện trong nhiều đề thi thử rồi, trong tuyển tập bộ 3 câu khó của VMF cũng xuất hiện

Đề bị nhầm 1 chút nha, sửa lại: $P= \frac{1}{2\sqrt{x^{2}+y^{2}+z^{2}-2(2x+y-3)}}-\frac{1}{y(x-1)(z+1)}$
 
Đặt $a=x-2$, $b=y-1$, $c=z$. Ta có a, b, c>0
$P=\frac{1}{2\sqrt{\sum a^{2}+1}}-\frac{1}{(a+1)(b+1)(c+1)}=\frac{1}{\sqrt{(1+1+1+1)(a^{2}+b^{2}+c^{2}+1)}}-\frac{1}{(a+1)(b+1)(c+1)}\leq \frac{1}{a+b+c+1}-\frac{27}{(a+1+b+1+c+1)^{3}}=\frac{1}{a+b+c+1}-\frac{27}{(a+b+c+3)^{3}}$
Đăt $t=a+b+c$ $(t>0)$, khi đó: $P=f(t)=\frac{1}{t+1}-\frac{27}{(t+3)^{3}}$
Có: $f'(t)=0\Leftrightarrow t=3\Rightarrow f(t)_{max}=f(3)=\frac{1}{8}$
Vậy $P\leq \frac{1}{8}$. Đẳng thức xảy ra khi x=3, y=2, z=1
 
P/s: sao mình thấy nhiều câu thi thử trông nó có vẻ đậm chất thi HSG quá vậy nhỉ @@ rồi một số lời giải các bạn thì ôi thôi rồi luôn, holder, schur chóng hết cả mặt...

Bài viết đã được chỉnh sửa nội dung bởi quanguefa: 17-04-2016 - 23:33

Xem topic "Chuyên đề các bài Toán lãi suất Casio" tại đây

 

:like Visit my facebook


#57
quanguefa

quanguefa

    Thiếu úy

  • Thành viên
  • 596 Bài viết

Bài 33: (lớp toán Lý Thái Tổ, thi thử lần thứ 14 năm 2015).

Cho x, y, z thực dương thỏa mãn $x^{2}+y^{2}+z^{2}=2$. Tìm GTNN của biểu thức

$P=\frac{3x^2+3y^2}{8}+\frac{2z}{x+y}-\frac{z^2+z}{(x+1)(y+1)}-\sqrt{3}z$


Bài viết đã được chỉnh sửa nội dung bởi quanguefa: 17-04-2016 - 23:41

Xem topic "Chuyên đề các bài Toán lãi suất Casio" tại đây

 

:like Visit my facebook


#58
quanguefa

quanguefa

    Thiếu úy

  • Thành viên
  • 596 Bài viết

Bài 24:Cho $x,y,z$ là các số thực không âm thỏa mãn $x^{2}+y^{2}+z^{2}=3$. Tìm GTNN của 

   $P=\frac{16}{\sqrt{x^{2}y^{2}+y^{2}z^{2}+z^{2}x^{2}+1}}+\frac{xy+yz+zx+1}{x+y+z}$

 

_Đề thi thử Chu Văn An Sơn La_

Bài 24:

Đầu tiên ta chứng minh: $\sum x\geq \sum x^2y^2$

Ta có: $\sum x^2y^2=\frac{(\sum x^2)^{2}-\sum x^{4}}{2}=\frac{9-\sum x^{4}}{2}$

Cần chứng minh: $\frac{9-\sum x^{4}}{2}\leq\sum  x\Leftrightarrow 2\sum x+\sum x^4\geq 9$

BĐT này đúng nhờ đánh giá AM-GM: $x^4+x+x\geq 3x^2$

Từ đó ta có: $P\geq \frac{16}{\sqrt{\sum x+1}}+\frac{\frac{(\sum x)^2-\sum x^2}{2}+1}{\sum x}=\frac{16}{\sqrt{\sum x+1}}+\frac{(\sum x)^2-1}{2\sum x}$

Đặt $t=x+y+z$ $(t\leq 3)$. Khi đó: $P\geq f(t)=\frac{16}{\sqrt{t+1}}+\frac{t^2-1}{2t}$

Tính đạo hàm suy ra f(t) nghịch biến, suy ra: $f(t)_{min}=f(3)=\frac{28}{3}\Rightarrow P\geq \frac{28}{3}$

Vậy.... dấu bằng xảy ra khi x=y=z=1

 

Sử dụng hơi nhiều dấu sigma các bạn thông cảm


Bài viết đã được chỉnh sửa nội dung bởi quanguefa: 18-04-2016 - 00:26

Xem topic "Chuyên đề các bài Toán lãi suất Casio" tại đây

 

:like Visit my facebook


#59
quanguefa

quanguefa

    Thiếu úy

  • Thành viên
  • 596 Bài viết

Bài 34:

Cho x, y, z thực dương: $x+y+1=z$

Tìm GTNN của: $P=\sum \frac{x^3}{x+yz}+\frac{14}{(z+1)\sqrt{(x+1)(y+1)}}$


Bài viết đã được chỉnh sửa nội dung bởi quanguefa: 19-04-2016 - 21:26
Lần sau ghi rõ nguồn

Xem topic "Chuyên đề các bài Toán lãi suất Casio" tại đây

 

:like Visit my facebook


#60
MarkNguyen

MarkNguyen

    Lính mới

  • Thành viên mới
  • 3 Bài viết

Bài 28: (Đề thi thử của sở GD- ĐT Thanh Hóa năm 2016)

Cho $a,b,c$ là độ dài ba cạnh của một tam giác. Tìm giá trị lớn nhất của biểu thức:

$P=\left ( a+b+c \right )\left ( \frac{3a-b}{a^{2}+ab}+\frac{3b-c}{b^{2}+bc}+\frac{3c-a}{c^{2}+ca} \right )$

 

Vì biểu thức P có tính thuần nhất nên chuẩn hóa $a+b+c=1$. Ta có:

$P= \frac{3a-b}{a^{2}+ab}+\frac{3b-c}{b^{2}+bc}+\frac{3c-a}{c^{2}+ca}$

$=\frac{4a-\left ( a+b \right )}{a\left ( a+b \right )}+\frac{4b-\left ( b+c \right )}{b\left ( b+c \right )}+\frac{4c-\left ( c+a \right )}{c\left ( c+a \right )}$

$=\frac{4}{a+b}-\frac{1}{a}+\frac{4}{b+c}-\frac{1}{b}+\frac{4}{c+a}-\frac{1}{c}$

Dự đoán GTLN của P=9. Ta sẽ chứng minh:

$\frac{4}{a+b}-\frac{1}{a}+\frac{4}{b+c}-\frac{1}{b}+\frac{4}{c+a}-\frac{1}{c}\leq 9$

$\Leftrightarrow 9+\frac{1}{a}+\frac{1}{b}+\frac{1}{c}\geq \frac{4}{a+b}+\frac{4}{b+c}+\frac{4}{c+a}$

$\Leftrightarrow 9+\left ( a+b+c \right )\left ( \frac{1}{a}+\frac{1}{b}+\frac{1}{c} \right )\geq \left ( a+b+c \right )\left ( \frac{4}{a+b}+\frac{4}{b+c}+\frac{4}{c+a} \right )$

$\Leftrightarrow a\left ( \frac{1}{b}+\frac{1}{c} \right )+b\left (\frac{1}{c}+\frac{1}{a} \right )+c\left ( \frac{1}{a}+\frac{1}{b} \right )\geq \frac{4a}{b+c}+\frac{4b}{c+a}+\frac{4c}{a+b}$ $\left ( * \right )$

Theo BĐT Cauchy-Schwarz, ta có

 $\frac{1}{b}+\frac{1}{c}\geq \frac{4}{b+c}\Rightarrow a\left ( \frac{1}{b}+\frac{1}{c} \right )\geq \frac{4a}{b+c}$

Tương tự, ta có:

$\left\{\begin{matrix} b\left ( \frac{1}{c}+\frac{1}{a} \right )\geq \frac{4b}{c+a}\\c\left ( \frac{1}{a}+\frac{1}{b} \right )\geq \frac{4c}{a+b} \end{matrix}\right.$

$\Rightarrow \left ( * \right )$  đúng

$\Rightarrow P\leq 9$

Vậy GTLN của P=9 $\Leftrightarrow a=b=c$

*Nhận xét, có lẽ người ra đề nghĩ ra bài toán trên dựa vào 1 bất đẳng thức quen thuộc: 

Cho a,b,c là độ dài 3 cạnh của 1 tam giác. Chứng minh rằng:

$9+\frac{1}{a}+\frac{1}{b}+\frac{1}{c}\geq \frac{4}{a+b}+\frac{4}{b+c}+\frac{4}{c+a}$






1 người đang xem chủ đề

0 thành viên, 1 khách, 0 thành viên ẩn danh